You are on page 1of 37

SECRETARY OF FINANCE vs.

ILARDE

Facts:

1. Cabaluna failed to pay land taxes. A breakdown of the computation of the delinquent
taxes and penalties on his lots and residential house as reflected in the various receipts
issued by the City Treasurer's Office of Iloilo City, shows that more than 24% of the
delinquent taxes were charged and collected by way of penalties.
2. Cabaluna paid his land taxes under protest.
3. He then filed a formal letter of protest with the City Treasurer of Iloilo City wherein he
contends that the City Treasurer's computation of penalties was erroneous since the
rate of penalty applied exceeded 24% in contravention of Section 66 of P.D. No. 464,
otherwise known as the Real Property Tax Code, as amended.
4. In response, however, Assistant City Treasurer Tulio, for and in behalf of the City
Treasurer of Iloilo, turned down private respondent's protest, citing Sec. 4(c) of
Joint Assessment Regulations No. 1-85 and Local Treasury Regulations No. 2-85
of the then Ministry (now Department) of Finance. which reads:

Sec. 4. Computation of Penalties on Delinquent Real Property Taxes.

(a) Unless condoned, wholly or partially, in a duly approved resolution of


the Local Sanggunian, delinquent real property taxes shall be subject to
penalty at the rate of 2% for every month of delinquency, provided that
the total penalty for one tax year shall not exceed 24%.

(b) Failure to pay on time at least the 1 st quarter installment of the real
property tax shall constitute a waiver on the part of the property owner
or administrator to avail of the privilege granted by law for him to pay
without penalty his annual realty tax obligation in 4 equal installment on
or before the end of every quarter of the tax year.

Accordingly, if the portion of the real property tax due for the 1 st quarter
of tax year is not paid on or before the 31 st day of March of the same
year, the penalty shall be reckoned from the 1 st day of January at the rate
of 2% for every month of delinquency on the basis of the total amount
due for the entire year and not only on the amount due for the said 1 st
quarter of the tax year.

(c) The penalty of 2% per month of delinquency, or 24% per annum, as


the case may be, shall continue to be imposed on the unpaid tax
from the time the delinquency was incurred up to the time that it
is paid for in full. (emphasis supplied)

5. Cabaluna filed a Petition for Declaratory Relief with Damages on 06 July 1993 before
the sala of respondent Judge, assailing Joint Assessment Regulations No. 1-85 and
Local Treasury Regulations No. 2-85 which, according to him, violated Section 66 of
P.D. No. 464 which fixed the maximum penalty for delinquency in the payment of real
estate taxes at 24% of the delinquent tax.

RTC: Section 4(c) of Joint Assessment Regulation No. 1-85 and Local Treasury Regulation
No. 2-85 are null and void. Penalty that should be imposed for delinquency in the payment
of real property taxes should be 2% on the amount of the delinquent tax for each month of
delinquency or fraction thereof, until the delinquent tax is fully paid but in no case shall the
total penalty exceed 24% of the delinquent tax as provided for in Section 66 of P.D. 464
otherwise known as the Real Property Tax Code.

Issue:

WON Joint Assessment Regulations No. 1-85 and Local Treasury Regulations No. 2-85 are
valid. NO.

Held:

The subject Regulations must be struck down for being repugnant to Section 66 of P.D. No.
464 or the Real Property Tax Code, which provides: That in no case shall the total penalty
exceed twenty-four per centum of the delinquent tax. The rate of penalty for tax
delinquency fixed herein shall be uniformly applied in all provinces and cities.

As adeptly observed by the trial court, the penalty imposed under the assailed Regulations
has no limit inasmuch as the 24% penalty per annum shall be continuously imposed on the
unpaid tax until it is paid for in full unlike that imposed under Section 66 of the Real
Property Tax Code where the total penalty is limited only to 24% percent of the delinquent
tax.

The secretary anchors his claim on E.O. No. 73 which provides that: "The Minister of Finance
shall promulgate the necessary rules and regulations to implement this Executive Order."
However, E.O. No. 73 did not touch at all on the topic of amendment of rates of delinquent
taxes or the amendment of rates of penalty on delinquent taxes. Neither did E.O. No. 1019
directly or indirectly vest upon the Department of Finance the right to fiddle with the rates
of penalty to be assessed on delinquency taxes as contained in the Real Property Tax Code.
Despite the promulgation of E.O. No. 73, P.D. No. 464 in general and Section 66 in
particular, remained to be good law. Furthermore, repeal by implication is not favored.

Assuming that E.O. No. 73 has authorized the petitioner to issue the objected Regulations,
such conferment of powers is void for being repugnant to the well-established doctrine in
political law that the power of taxation is generally vested with the legislature. The
power delegated to the executive branch, in this case the Ministry of Finance, to lay down
implementing rules must, nevertheless, be germane to the general law it seeks to apply.
The implementing rules cannot add to or detract from the provisions of the law it is
designed to implement.

The fact that private respondent Cabaluna was responsible for the Memorandum which
implemented the assailed regulations does not put him in estoppel from seeking the
nullification of said Regulations.
BENGUET CORPORATION vs. CBAA
210 SCRA 579, June 29, 1992

Facts:
The Provincial Assessor of Benguet, through the Municipal Assessor of Itogon,
assessed real property tax on the bunkhouses of petitioner Benguet Corporation occupied
for residential purposes by its rank and file employees under Tax Declarations Nos. 8471
(effective 1985) and 10454 (effective 1986). According to the Provincial Assessor, the tax
exemption of bunkhouses under Sec. 3 (a), P.D. 745 (Liberalizing the Financing and Credit
Terms for Low Cost Housing Projects of Domestic Corporations and Partnerships), was
withdrawn by P.D. 1955 (Withdrawing, Subject to Certain Conditions, the Duty and Tax
Privileges Granted to Private Business Enterprises and/or Persons Engaged in Any Economic
Activity, and Other Purposes). Petitioner appealed the assessment on Tax Declarations Nos.
8471 and 10454 to the Local Board of Assessment Appeals (LBAA) of the Province of
Benguet, docketed as LBAA Cases Nos. 42 and 43, respectively. Both were heard jointly.

Meanwhile, the parties agreed to suspend hearings in LBAA Cases Nos. 42 and 43 to
await the outcome of another case, LBAA Case No. 41, covering Tax Declaration No.
9534(effective 1984), which involved the same parties and issue until the appeal was
decided by the Central Board of Assessment Appeals (CBAA). On July 15, 1986, CBAA
handed down its decision in LBAA Case No. 41 holding that the buildings of petitioner used
as dwellings by its rank and file employees were exempt from real property tax pursuant to
P.D. 745.

Thereafter, the proceedings in LBAA Cases Nos. 42 and 43 proceeded after which a
decision was rendered affirming the taxability of subject property of petitioner. On appeal,
CBAA sustained the decision holding that the realty tax exemption under P.D. 745 was
withdrawn by P.D. 1955 and E.O. 93, so that petitioner should have applied for restoration
of the exemption with the Fiscal Incentives Review Board (FIRB). Petitioner contends that
local government units are without any authority to levy realty taxes on mines pursuant to
Sec. 52 of P.D. 463 and Sec. 5 (m) of The Local Tax Code, as amended by P.D. 426
(reiterated in Secs. 17 [d] and 22 [c], same Code).

Issue:

W/N the LGU may validly assess real property tax on the properties of petitioner
considering the prohibition in The Local Tax Code (P.D. 231) and the Mineral Resources
Development Decree of 1974 (P.D. 463) against imposition of taxes on mines by local
governments.

Ruling:

YES. Aside from the fact that the petitioners are estopped from estopped from
raising the question of lack of authority to issue the challenged assessments inasmuch as it
was never raised before, hence, not passed upon by, the municipal and provincial assessors,
LBAA and CBAA, the provisions of Sec. 52 of the Mineral Resources Development Decree of
1974 (P.D. 463), and Secs. 5 (m), 17 (d) and 22 (c) of The Local Tax Code (P.D. 231) cited
by petitioner are mere limitations on the taxing power of local government units; they are
not pertinent to the issue before Us and, therefore, cannot and should not affect the
imposition of real property tax by the national government.
While local government units are charged with fixing the rate of real property taxes,
it does not necessarily follow from that authority the determination of whether or not to
impose the tax. In fact, local governments have no alternative but to collect taxes as
mandated in Sec. 38 of The Real Property Tax Code, which states:

Sec. 38. Incidence of Real Property TaxThere shall be levied, assessed and
collected in all provinces, cities and municipalities an annual ad valorem tax
on real property, such as land, buildings, machinery and other improvements
affixed or attached to real property not hereinafter specifically exempted.

It is thus clear from the foregoing that it is the national government, expressing itself
through the legislative branch that levies the real property tax. Consequently, when local
governments are required to fix the rates, they are merely constituted as agents of the
national government in the enforcement of the Real Property Tax Code. The delegation of
taxing power is not even involved here because the national government has already
imposed realty tax in Sec. 38 above-quoted, leaving only the enforcement to be done by
local governments.

The challenge of petitioner against the applicability of Meralco Securities Industrial


Corporation v. Central Board of Assessment Appeals, et al.,3 is unavailing, absent any
cogent reason to overturn the same. ThusMeralco Securities argues that the realty tax is
a local tax or levy and not a tax of general application. This argument is untenable because
the realty tax has always been imposed by the lawmaking body and later by the President of
the Philippines in the exercise of his lawmaking powers, as shown in Sections 342 et seq. of
the Revised Administrative Code, Act No. 3995, Commonwealth Act No. 470 and Presidential
Decree No. 464.

The realty tax is enforced throughout the Philippines and not merely in a particular
municipality or city but the proceeds of the tax accrue to the province, city, municipality and
barrio where the realty taxed is situated (Sec. 86, P.D. No. 464). In contrast, a local tax is
imposed by the municipal or city council by virtue of the Local Tax Code, Presidential Decree
No. 231, which took effect on July 1, 1973 (69 O.G. 6197).
NATIONAL DEVELOPMENT COMPANY vs. CEBU CITY AND AUGUSTO PACIS AS
TREASURER OF CEBU CITY,

G.R. No. 51593 November 5, 1992

Facts:

National Development Company (NDC) is a government-owned or controlled corporation


(GOCC) existing by virtue of C.A. 182 and E.O. 399. It operates subsidiary corporations one
of which is the National Warehousing Corporation (NWC). On August 10, 1939, the
President issued Proclamation No. 430 reserving Block no. 4, Reclamation Area No. 4, of
Cebu City, consisting of 4,599 square meters, for warehousing purposes under the
administration of NWC. Subsequently, a warehouse was constructed thereon. E.O. 93 was
passed dissolving NWC with NDC taking over its assets and functions.

Commencing 1948, Cebu City assessed and collected from NDC real estate taxes on the
land and the warehouse thereon. NDC paid the assessed real estate taxes under protest.
NDC wrote the City Assessor demanding full refund of the real estate taxes paid to Cebu
City claiming that the land and the warehouse standing thereon belonged to the Republic
and therefore exempt from taxation. Cebu City did not acquiesce in the demand, hence
NDC institute an action before CFI of Manila. The court ruled in favor of NDC. Cebu City
appealed the case before the CA which was later on certified to the SC as one involving pure
questions of law.

Issue:

Whether NDC is exempted from payment of the real estate taxes on the land reserved by
the President for warehousing purposes as well as the warehouse constructed thereon?

Arguments:

CEBU insists on taxability of the subject properties, claiming that no law grants NDC
exemption from real estate taxes, and that NDC, as recipient of the land reserved by the
President pursuant to Sec. 83 of the Public Land Act, is liable for payment or ordinary (real
estate) taxes under Sec. 115 therefore. CEBU contends that the properties have ceased to
be tax exempt under the Assessment Law when the government disposed of them in favor
of NDC.

On the other hand, NDC maintains the Sec. 3 of the Assessment Law, which exempts
properties owned by the Republic from real estate tax, includes subject properties in the
exemption. It invokes the ruling in Board of Assessment Appeals vs. CTA & NWSA which
held that properties of NWSA, a GOCC, were exempt from real estate tax because Sec. 3 of
the Assessment Law applied to all government properties whether held in governmental or
proprietary capacity. NDC rejects the applicability of Sec. 115 of the Public Land Act to the
subject land, claiming that provision contemplates dispositions of public land with eventual
transfer of title. In addition, NDC believes that it is neither a grantee of a public land nor an
applicant within the purview of the same provision.
Ruling: One of the principal issues is the interpretation of a provision of the Assessment
Law, the precursor of the then Real Property Tax Code and the Local Government Code,
where "ownership" of the property and not "use" is the test of tax liability.

Section, 3 par. (a), of the Assessment Law, on which NDC claims real estate tax exemption,
provides

Section 3. Property exempt from tax. The exemptions shall be as follows:


(a) Property owned by the United States of America, the Commonwealth of
the Philippines, any province, city, municipality at municipal district . . .

To come within the ambit of the exemption provided in Art. 3, par. (a), of the Assessment
Law, it is important to establish that the property is owned by the government or its
unincorporated agency, and once government ownership is determined, the nature of the
use of the property, whether for proprietary or sovereign purposes, becomes immaterial.
What appears to have been ceded to NWC (later transferred to NDC) is merely the
administration of the property while the government retains ownership of what has been
declared reserved for warehousing purposes under Proclamation No. 430.

A reserved land is defined as a "public land that has been withheld or kept back from sale or
disposition." The land remains "absolute property of the government." The government
"does not part with its title by reserving them (lands), but simply gives notice to all the
world that it desires them for a certain purpose." Absolute disposition of land is not implied
from reservation; it merely means "a withdrawal of a specified portion of the public domain
from disposal under the land laws and the appropriation thereof, for the time being, to some
particular use or purpose of the general government." As its title remains with the Republic,
the reserved land is clearly recovered by the tax exemption provision.

CEBU contends that the reservation of the property in favor of NWC or NDC is a form of
disposition of public land which, subjects the recipient (NDC) to real estate taxation under
Sec. 115 of the Public Land Act. as amended by R.A. 436. However, the court viewed that
the effect of reservation under Sec. 83 is to segregate a piece of public land and transform
it into non-alienable or non-disposable under the Public Land Act. Section 115, on the other
hand, applies to disposable public lands. Clearly, therefore, Sec. 115 does not apply to lands
reserved under Sec. 83. Consequently, the subject reserved public land remains tax exempt.

However, as regards the warehouse constructed on a public reservation, a different rule


should apply because "the exemption of public property from taxation does not extend to
improvements on the public lands made by pre-emptioners, homesteaders and other
claimants, or occupants, at their own expense, and these are taxable by the state . . ."
Consequently, the warehouse constructed on the reserved land by NWC (now under
administration by NDC), indeed, should properly be assessed real estate tax as such
improvement does not appear to belong to the Republic.

Since the reservation is exempt from realty tax, the erroneous tax payments collected by
CEBU should be refunded to NDC. This is in consonance with Sec. 40, par. (a) of the former
Real Property Tax Code which exempted from taxation real property owned by the Republic
of the Philippines or any of its political subdivisions, as well as any GOCC so exempt by its
charter.
PROVINCE OF TARLAC vs. ALCANTARA

Facts:
Tarlac Provincial Treasurer filed a complaint against Tarlac Enterprises, Inc., a
corporation that owns several parcels of land in which the former declared for purposes of
taxation. Petitioner prayed that private respondent be ordered to pay the sum of
P532,435.55 representing the accrued real estates taxes, as well as damages and the costs
of the suit. The private respondent filed its answer admitting that demands for the payment
of real property taxes had been made by the petitioner but it refused to pay the same for
the reason that under Sec. 40, paragraph (g) of Presidential Decree No. 464 in relation to
P.D. No. 551, as amended, it was exempt from paying said tax.
On August 12, 1983, the lower court rendered the decision dismissing the complaint. It
ruled the P.D. No. 551 expressly exempts private respondent from paying the real property
taxes demanded, it being a grantee of a franchise to generate, distribute and sell electric
current for light. The court held that in lieu of said taxes, private respondent had been
required to pay two percent (2%) franchise tax in line with the intent of the law to give
assistance to operators such as the private respondent to enable the consumers to enjoy
cheaper rates. Citing the case of Butuan Sawmill, Inc. v. City of Butuan, the court ruled that
local governments are without power to tax the electric companies already subject to
franchise tax unless their franchise allows the imposition of additional tax.
Petitioner contends that respondent judge erred in: (a) holding that private respondent is
exempt from the payment of realty tax under P.D. No. 551, as amended; and (b) ruling,
under the authority of Butuan Sawmill, Inc. v. Butuan City, that it is without power to
impose said realty tax on private respondent.
Sec. 40(g) of P.D. No. 464, the Real Property Tax Code, provides:

SEC. 40. Exemptions from Real Property Tax. The exemption shall be as
follows:

xxx xxx xxx

(g) Real property exempt under other laws.

Private respondent contends that the "other laws" referred to in this Section is P.D. No. 551
(Lowering the Cost to Consumer of Electricity by Reducing the Franchise Tax Payable by
Electric Franchise Holders and the Tariff on Fuel Oils for the Generation of Electric Power by
Public Utilities). Its pertinent provisions state:

SECTION 1. Any provision of law or local ordinance to the contrary


notwithstanding, the franchise tax payable by all grantees of franchises to
generate, distribute and sell electric current for light, heat and power shall be
two (2%) of their gross receipts received from the sale of electric current and
from transactions incident to the generation, distribution and sale of electric
current.

Such franchise tax shall be payable to the Commissioner of Internal Revenue


or his duly authorized representative on or before the twentieth day of the
month following the end of each calendar quarter or month as may be
provided in the respective franchise or pertinent municipal regulation and
shall, any provision of the Local Tax Code or any other law to the contrary
notwithstanding, be in lieu of all taxes and assessments of whatever
nature imposed by any national or local authority on earnings,
receipts, income and privilege of generation, distribution and sale of
electric current. 9 (Emphasis supplied)

P.D. NO. 551 was amended on December 19, 1975 by P.D. No. 852 with the insertion of the
phrase "and for the manufacture, distribution and sale of city gas" between the phrases "...
light, heat and power" and "shall be two (2%) ..."

Issue:

Whether or not private respondent Tarlac Enterprises, Inc. is exempt from the payment of
real property tax under Sec. 40 (g) of P.D. No. 464 in relation to P.D. No. 551, as amended.

Ruling: NO.

The court did not agree with the lower court that the phrase "in lieu of all taxes and
assessments of whatever nature" in the second paragraph of Sec. 1 of P.D. No. 551
expressly exempts private respondent from paying real property taxes. The said proviso is
modified and delimited by the phrase "on earnings, receipts, income and privilege of
generation, distribution and sale" which specifies the kinds of taxes and assessments which
shall not be collected in view of the imposition of the franchise tax. Said enumerated items
upon which taxes shall not be imposed, have no relation at all to, and are entirely different
form, real properties subject to tax.
If the intention of the law is to exempt electric franchise grantees from paying real property
tax and to make the two (2%) percent franchise tax the only imposable tax, then said
enumerated items would not have been added when P.D. No. 852 was enacted to amend
P.D. No. 551. The legislative authority instead went on to enumerate what should not be
subject to tax thereby delimiting the extent of the exemption.
We also find misplaced the lower court's and the private respondent's reliance on Butuan
Sawmill., Inc. v. City of Butuan. In that case, the questioned tax is a tax on the gross sales
or receipts of said sawmill while the tax involved herein is a real property tax. The City of
Butuan is categorically prohibited therein by Sec. 2(j) of Local Autonomy Act from imposing
"taxes of any kind ... on person paying franchise tax." On the other hand, P.D. No. 551 is
not as all-encompassing as said provision of the Local Autonomy Act for it enumerates the
items which are not taxable by virtue of the payment of franchise tax.
AQUILINO PIMENTEL vs. AGUIRRE
G.R. No. 132988, July 19, 2000

Facts

On December, 1997, the President issued AO 372 (Adoption of Economy Measures in


Government for FY 1998). The AO provided that (a) 10% of the Internal Revenue allotment
to LGUs is withheld. Further it (b) "directs" LGUs to reduce their expenditures by 25 percent
Subsequently, on December 10, 1998, President Estrada issued AO 43, amending Section 4
of AO 372, by reducing to five percent (5%) the amount of internal revenue allotment (IRA)
to be withheld from the LGUs.

Petitioner contends that by issuing AO 372, the President exercised the power of control
over LGUs in contravention of law. Moreover, withholding 10% of the IRA is in contravention
of Sec 286 LGC and of Sec 6 Article X of the Constitution, providing for the automatic
release to each of these units its share in the national internal revenue.
The Solicitor General, on the other hand, argues that the aforesaid AO was purportedly in
order to cope with the nations economic difficulties brought about by the peso depreciation
on that said period. Further, he claims that AO 372 was issued merely as an exercise of the
Presidents power of supervision over LGUs. It allegedly does not violate local fiscal
autonomy, because it merely directs local governments to identify measures that will reduce
their total expenditures for non-personal services by at least 25 percent. Likewise, the
withholding of 10 percent of the LGUs IRA does not violate the statutory prohibition on the
imposition of any lien or holdback on their revenue shares, because such withholding is
"temporary in nature pending the assessment and evaluation by the Development
Coordination Committee of the emerging fiscal situation."

Issues:
1 W/N Section 1 of AO 372, insofar as it "directs" LGUs to reduce their expenditures by
25 percent is a valid exercise of the President's power of general supervision over local
governments.
2 W/N Section 4 of AO 372, which withholds 10 percent of their internal revenue
allotments, are valid exercises of the President's power of general supervision over local
governments.

Ruling

1 YES. There are several requisites before the President may interfere in local fiscal
matters: (1) an unmanaged public sector deficit of the national government; (2)
consultations with the presiding officers of the Senate and the House of Representatives and
the presidents of the various local leagues; and (3) the corresponding recommendation of
the secretaries of the Department of Finance, Interior and Local Government, and Budget
and Management. Furthermore, any adjustment in the allotment shall in no case be less
than thirty percent (30%) of the collection of national internal revenue taxes of the third
fiscal year preceding the current one.1

1 284 (c) of the Local Government Code.


Petitioner points out that respondents failed to comply with the above requisites before the
issuance and the implementation of AO 372. At the very least, the respondents did not
even try to show that the national government was suffering from an unmanageable public
sector deficit. Neither did they claim having conducted consultations with the different
leagues of local governments. Without these requisites, the President has no authority to
adjust, much less to reduce, unilaterally the LGU's internal revenue allotment.

Although the Supreme Court agrees with the Petitioner that the requisites were not
complied with, it still holds that the Presidents directive in AO 372 is in conformity with law,
and does constitute interference to local autonomy. There is interference if Section 1 of AO
372 was couched in mandatory or binding language. While the wordings of Section 1 of
AO 3722 have a rather commanding tone, the provision is merely an advisory to prevail
upon local executives to recognize the need for fiscal restraint in a period of economic
difficulty. Indeed, all concerned would do well to heed the President's call to unity, solidarity
and teamwork to help alleviate the crisis. It is understood, however, that no legal sanction
may be imposed upon LGUs and their officials who do not follow such advice.

2 NO. A basic feature of local fiscal autonomy is the automatic release of the shares of LGUs
in the national internal revenue as mandated by the Constitution. The Local Government
Code. specifies further that the release shall be made directly to the LGU concerned within
five (5) days after every quarter of the year and "shall not be subject to any lien or
holdback that may be imposed by the national government for whatever purpose.

The use of the term "shall" shows that the provision is imperative. Therefore, Section 4 of
AO 372, which orders the withholding of 10 percent of the LGUs' IRA "pending the
assessment and evaluation by the Development Budget Coordinating Committee of the
emerging fiscal situation" in the country clearly contravenes the Constitution and the
law. Although temporary, it is equivalent to a holdback, which means "something held back
or withheld, often temporarily. Hence, the "temporary" nature of the retention by the
national government does not matter. Any retention is prohibited. Therefore, the President
clearly overstepped the bounds of his lawful authority when he issued Section 4 of AO
372.

2 The above Section states that (LGUs must) "identify and implement measures x x x that will reduce total
expenditures x x x by at least 25% of authorized regular appropriation."
ESTATE OF GREGORIA FRANCISCO vs. COURT OF APPEALS

FACTS:
1 A Quonset (lightweight structure) was constructed by American Liberation Forces in
1944. In 1946, it was purchased by Gregoria Francisco.
2 The Quonset stands on a lot owned by the Philippine Ports Authority (PPA).
3 By virtue of Proclamation No. 83 issued by President Quirino, the said lot was
declared for the exclusive use of port facilities.
4 The PPA then issued to Tan Gin San (spouse of Gregoria Francisco) a permit to
occupy the lot where the building stands for a period of 1 year to expire on 31
December 1989. Tan Gin San used the Quonset for the storage of copra

5 In May 1989, the Mayor notified Tan Gin San to remove or relocate its Quonset
structure citing Ordinance No. 147, noting its antiquated and dilapidated structure;
and stressing the clean-up campaign on illegal squatters and unsanitary
surroundings along Strong Boulevard.
6 Likewise, according to the Mayor, the building was outside the warehouse zone;
hence, it was a non-confirming structure.
7 The Mayors notification remained unheeded; thus, the Mayor ordered its demolition
on 24 May 1989.
8 Petitioner then filed a Writ of Prohibition with Injunction and Damages to the RTC
a RTC: denied petitioners petition and upheld the power of the Mayor to order the
demolition
b CA: reversed RTC decision. The Mayor was not vested with power to order
summarily without any judicial proceeding to demolish the Quonset building
which was not a nuisance per se.
c CA upon reconsideration: reversed itself.
9 The Quonset was completely demolished on 06 September 1989.

ISSUE/HELD:

Whether or not it was proper for the Mayor to summarily, without judicial process, order the
demolition of the non-conforming structure. NO.

RATIO:

It is not disputed that the Quonset building, which is being used for the storage of copra, is
located outside the zone for warehouses. It is referred to in Ordinance as a non-conforming
structure, which should be relocated. In the event that an immediate relocation of the
building cannot be accomplished, Section 16 of Ordinance No. 147 provides, A
certificate of non-conformance for all non-conforming uses shall be applied for the by the
owner or agent of the property involved within 12 months from the approval of this
Ordinance, otherwise the non-conforming use may be condemned or removed at the
owners expense.

Assuming that petitioner failed to apply for a Certificate of Non-conformance, the provision
should not be interpreted as authorizing the summary removal of a non-conforming building
by the municipal government. For if it does, it must be struck down for being in
contravention of the requirements of due process, as originally held by the CA.
The enforcement and administration of the provisions of the Ordinance resides with the
Zoning Administrator, who may call upon the City Fiscal to institute the necessary legal
proceedings to enforce the provisions of the Ordinance. Violation of a municipal ordinance
does not empower the Mayor to avail of extra-judicial remedies. ON the contrary, the Local
Government Code imposes upon him the duty to cause to be instituted judicial proceedings
in connection with the violation of ordinances.

Respondents cannot seek cover under the General Welfare Clause authorizing the
abatement of nuisances without judicial proceedings. That tenet applies to a nuisance per se
or one which affects the immediate safety of persons and property and may be summarily
abated under the undefined law of necessity.

The storage of copra in the Quonset building is a legitimate business. By its nature, it
cannot be said to be injurious to the rights of property, health or of comfort of the
community. If it be a nuisance per accidens it may be so proven in a hearing conducted for
that purpose. It is not per se a nuisance warranting its summary abatement without judicial
intervention.

While the Sanggunian may provide for the abatement of nuisance, it cannot declare a
particular thing as a nuisance per se and order its condemnation. The nuisance can only be
so adjudged by judicial determination.

Petitioner was in lawful possession of the lot and Quonset building by virtue of the permit to
occupy issued by the PPA. It was not squatting on a public land. It was entitled to impartial
hearing before a tribunal who will decide whether the Quonset building constitute a nuisance
in law. There was no compelling reason for immediate demolition.

The public officials of Isabela, Basilan, exceeded their authority in abating summarily
petitioners Quonset building, without affording them due process of law. Thus, petitioners
are entitled to just compensation but not to damages.
TECHNOLOGY DEVELOPMENT vs. CA
GR. 94759, January 21, 1991

Facts: On February 16, 1989, petitioner, a domestic private corporation engaged in the
manufacture and export of charcoal briquette, received a letter from acting mayor Pablo N.
Cruz, ordering the full cessation of the operation of their located at Guyong, Sta. Maria,
Bulacan, until further order. The letter likewise requested Plant Manager Mr. Armando
Manese to bring with him to the office of the mayor on February 20, 1989 the following: a)
Building permit; b) Mayor's permit; c) Region III-Pollution of Environment and Natural
Resources Anti- Pollution Permit; and of other document.
At the requested conference, petitioner, through its representative, undertook to comply
with the request for the production of the required documents. In compliance with said
undertaking, petitioner commenced to secure "Region III-Department of Environmental and
Natural Resources Anti-Pollution Permit," although among the permits previously secured
prior to the operation of petitioner's plant was a "Temporary Permit to Operate Air Pollution
Installation" issued by the then National Pollution Control Commission (now Environmental
Management Bureau) and is now at a stage where the Environmental Management Bureau
is trying to determine the correct kind of anti-pollution devise to be installed as part of
petitioner's request for the renewal of its permit. Petitioner also sent its representatives to
the office of the mayor to secure a mayors permit however the request was not
entertained.

Subsequently, on April 6, 1989, without previous and reasonable notice upon petitioner, the
acting mayor ordered the Municipality's station commander to padlock the premises of
petitioner's plant, thus effectively causing the stoppage of its operation. As a result
petitioner filed instituted an action for certiorari, prohibition, mandamus with preliminary
injunction against the acting mayor. The trial court issued a writ of preliminary injunction
but the same was dissolved upon appeal of the acting mayor. Among the evidence
presented were an investigation report with findings that the fumes coming from the factory
may contain particles that are hazardous to the health of the people, 2 sheets of paper
containing signatures of the residents of Brgy. Guyong and a letter of complaint address to
the Governor.

Issue: W/N the mayor is acting within his powers when he ordered the closure of th factory.

Ruling: YES. While it is true that the matter of determining whether there is a pollution of
the environment that requires control if not prohibition of the operation of a business is
essentially addressed to the then National Pollution Control Commission of the Ministry of
Human Settlements, now the Environmental Management Bureau of the Department of
Environment and Natural Resources, it must be recognized that the mayor of a town has as
much responsibility to protect its inhabitants from pollution, and by virtue of his police
power, he may deny the application for a permit to operate a business or otherwise close
the same unless appropriate measures are taken to control and/or avoid injury to the health
of the residents of the community from the emissions in the operation of the business.

The Acting Mayor, in a letter of February 16, 1989, called the attention of petitioner
to the pollution emitted by the fumes of its plant whose offensive odor "not only pollute the
air in the locality but also affect the health of the residents in the area," so that petitioner
was ordered to stop its operation until further orders and it was required to bring the
following: (1) Building permit;(2) Mayor's permit; and(3) Region III-Department of
Environment and Natural Resources Anti-Pollution permit. This action of the Acting Mayor
was in response to the complaint of the residents of Barangay Guyong, Sta. Maria, Bulacan,
directed to the Provincial Governor through channels.

LAGUNA LAKE DEVELOPMENT AUTHORITY vs. COURT OF APPEALS

G.R. Nos. 120865-71 December 7, 1995

Facts: Towards environmental protection and ecology, navigational safety, and sustainable
development, Republic Act No. 4850 created the "Laguna Lake Development Authority."
Presidential Decree No. 813 of former President Ferdinand E. Marcos amended certain
sections of Republic Act No. 4850 because of the concern for the rapid expansion of
Metropolitan Manila, the suburbs and the lakeshore towns of Laguna de Bay, combined with
current and prospective uses of the lake for municipal-industrial water supply, irrigation,
fisheries, and the like. As such, Laguna Lake Development Authority was endowed with
special powers one of which is to have exclusive jurisdiction to issue new permit for the use
of the lake waters for any projects or activities in or affecting the said lake including
navigation, construction, and operation of fishpens, fish enclosures, fish corrals and the like
for the purpose of effectively regulating and monitoring activities in Laguna de Bay.

To more effectively perform the role of the Authority under Republic Act No. 4850, the Chief
Executive enacted Executive Order No. 927 to further define and enlarge the functions and
powers of the Authority and named and enumerated the towns, cities and provinces
encompassed by the term "Laguna de Bay Region". Also, Section 3 of Executive Order No.
927 provided for the proper sharing of fees collected between the LLDA and local
government units concerned.

Then Republic Act No. 7160, the Local Government Code of 1991, was enacted. The
municipalities in the Laguna Lake Region interpreted the provisions of this law to mean that
the newly passed law gave municipal governments the exclusive jurisdiction to issue fishing
privileges within their municipal waters because R.A. 7160 provides:

Sec. 149. Fishery Rentals, Fees and Charges.

(a) Municipalities shall have the exclusive authority to grant fishery privileges in the
municipal waters and impose rental fees or charges therefor in accordance with the
provisions of this Section.

Municipal governments thereupon assumed the authority to issue fishing privileges and
fishpen permits. Unregulated fishpens and fishcages occupied almost one-third of the entire
lake water surface area. The implementation by the lakeshore municipalities of separate
independent policies in the operation of fishpens and fishcages within their claimed
territorial municipal waters in the lake and their indiscriminate grant of fishpen permits have
already saturated the lake area with fishpens, thereby aggravating the current
environmental problems and ecological stress of Laguna Lake.

In view of the foregoing circumstances, the LLDA served notice to the general public that all
fishpens, fishcages and other aqua-culture structures in the Laguna de Bay Region, which
were not registered or to which no application for registration and/or permit has been filed
with LLDA were outright illegal. LLDA also instructed the concerned owners of the illegally
constructed fishpens, fishcages and other aqua-culture structures to dismantle the
structures, otherwise, demolition shall be effected.

Reacting thereto, the affected fishpen owners filed injunction cases against the LLDA before
various regional trial courts. Temporary restraining order/writs of preliminary mandatory
injunction were issued enjoining the LLDA from demolishing the fishpens and similar
structures. Hence, a petition for certiorari, prohibition and injunction were filed by the LLDA
with the Supreme Court praying that LLDA be declared to be the exclusive authority to issue
said permits.

Issue: Which agency of the Government the Laguna Lake Development Authority or the
towns and municipalities comprising the region should exercise jurisdiction over the
Laguna Lake and its environs insofar as the issuance of permits for fishery privileges is
concerned?

Ruling: Section 4 (k) of the charter of the Laguna Lake Development Authority, Republic
Act No. 4850, the provisions of Presidential Decree No. 813, and Section 2 of Executive
Order No. 927, cited above, specifically provide that the Laguna Lake Development
Authority shall have exclusive jurisdiction to issue permits for the use of all surface water for
any projects or activities in or affecting the said region, including navigation, construction,
and operation of fishpens, fish enclosures, fish corrals and the like. On the other hand,
Republic Act No. 7160, the Local Government Code of 1991, has granted to the
municipalities the exclusive authority to grant fishery privileges in municipal waters. The
Sangguniang Bayan may grant fishery privileges to erect fish corrals, oyster, mussels or
other aquatic beds or bangus fry area within a definite zone of the municipal waters.

The provisions of Republic Act No. 7160 do not necessarily repeal the aforementioned laws
creating the Laguna Lake Development Authority and granting the latter water rights
authority over Laguna de Bay and the lake region. The Local Government Code of 1991 does
not contain any express provision which categorically expressly repeal the charter of the
Authority. It has to be conceded that there was no intent on the part of the legislature to
repeal Republic Act No. 4850 and its amendments. The repeal of laws should be made clear
and expressed.

It has to be conceded that the charter of the Laguna Lake Development Authority
constitutes a special law. Republic Act No. 7160, the Local Government Code of 1991, is a
general law. It is basic in statutory construction that the enactment of a later legislation
which is a general law cannot be construed to have repealed a special law. It is a well-
settled rule in this jurisdiction that "a special statute, provided for a particular case or class
of cases, is not repealed by a subsequent statute, general in its terms, provisions and
application, unless the intent to repeal or alter is manifest, although the terms of the
general law are broad enough to include the cases embraced in the special law." Thus, it has
to be concluded that the charter of the Authority should prevail over the Local Government
Code of 1991.

The power of the local government units to issue fishing privileges was clearly granted for
revenue purposes. This is evident from the fact that Section 149 of the New Local
Government Code empowering local governments to issue fishing permits is embodied in
Chapter 2, Book II, of Republic Act No. 7160 under the heading, "Specific Provisions On The
Taxing And Other Revenue Raising Power Of Local Government Units."
On the other hand, the power of the Authority to grant permits for fishpens, fishcages and
other aqua-culture structures is for the purpose of effectively regulating and monitoring
activities in the Laguna de Bay region (Section 2, Executive Order No. 927) and for lake
quality control and management. It does partake of the nature of police power which is the
most pervasive, the least limitable and the most demanding of all State powers including
the power of taxation. Accordingly, the charter of the Authority which embodies a valid
exercise of police power should prevail over the Local Government Code of 1991 on matters
affecting Laguna de Bay.

MODAY VS. CA

FACTS
The Sangguniang Bayan of the Municipality of Bunawan in Agusan del Sur passed Resolution
No. 43-89, authorizing the Municipal Mayor to Initiate the Petition for Expropriation of a One
(1) Hectare Portion of a lot owned by Percival Moday for the Site of Bunawan Farmers
Center and Other Government Sports Facilities.

In due time, Resolution No. 43-89 was approved by then Municipal Mayor Anuncio C.
Bustillo and transmitted to the Sangguniang Panlalawigan for its approval. On September
11, 1989, the Sangguniang Panlalawigan disapproved said Resolution and returned it with
the comment that "expropriation is unnecessary considering that there are still available lots
in Bunawan for the establishment of the government center."

The Municipality of Bunawan, herein public respondent, subsequently filed a petition for
Eminent Domain against petitioner Percival Moday before the Regional Trial Court.
Despite petitioners' opposition and after a hearing on the merits, the Regional Trial Court
granted the municipality's motion to take possession of the land. The lower court held that
the Sangguniang Panlalawigan's failure to declare the resolution invalid leaves it effective. It
added that the duty of the Sangguniang Panlalawigan is merely to review the ordinances
and resolutions passed by the Sangguniang Bayan under Section 208 (1) of B.P. Blg. 337,
old Local Government Code and that the exercise of eminent domain is not one of the two
acts enumerated in Section 19 thereof requiring the approval of the Sangguniang
Panlalawigan.

The Court of Appeals held that the public purpose for the expropriation is clear from
Resolution No. 43-89 and that since the Sangguniang Panlalawigan of Agusan del Sur did
not declare Resolution No. 43-89 invalid, expropriation of petitioners' property could
proceed.

ISSUE:
Whether or not the Municipality of Bunawan has the authority to expropriate the Lot
owned by Moday.

RULING: NO.
The Court finds no merit in the petition and affirms the decision of the Court of Appeals.

The Municipality of Bunawan's power to exercise the right of eminent domain is not disputed
as it is expressly provided for in Batas Pambansa Blg. 337, the local Government Code in
force at the time expropriation proceedings were initiated. Section 9 of said law states:
Sec. 9. Eminent Domain. A local government unit may, through its head and
acting pursuant to a resolution of its sanggunian, exercise the right of eminent
domain and institute condemnation proceedings for public use or purpose.

What petitioners question is the lack of authority of the municipality to exercise this right
since the Sangguniang Panlalawigan disapproved Resolution No. 43-89.

Section 153 of B.P. Blg. 337 provides:

Sec. 153. Sangguniang Panlalawigan Review. (1) Within thirty days after receiving
copies of approved ordinances, resolutions and executive orders promulgated by the
municipal mayor, the sangguniang panlalawigan shall examine the documents or
transmit them to the provincial attorney, or if there be none, to the provincial fiscal,
who shall examine them promptly and inform the sangguniang panlalawigan in
writing of any defect or impropriety which he may discover therein and make such
comments or recommendations as shall appear to him proper.

(2) If the sangguniang panlalawigan shall find that any municipal ordinance,
resolution or executive order is beyond the power conferred upon the sangguniang
bayan or the mayor, it shall declare such ordinance, resolution or executive order
invalid in whole or in part, entering its actions upon the minutes and advising the
proper municipal authorities thereof. The effect of such an action shall be to annul
the ordinance, resolution or executive order in question in whole or in part. The
action of the sangguniang panlalawigan shall be final.

xxx xxx xxx (Emphasis supplied.)

The Sangguniang Panlalawigan's disapproval of Municipal Resolution No. 43-89 is an infirm


action which does not render said resolution null and void. The law, as expressed in Section
153 of B.P. Blg. 337, grants the Sangguniang Panlalawigan the power to declare a municipal
resolution invalid on the sole ground that it is beyond the power of the Sangguniang Bayan
or the Mayor to issue. Although pertaining to a similar provision of law but different factual
milieu then obtaining, the Court's pronouncements in Velazco v. Blas, where we cited
significant early jurisprudence, are applicable to the case at bar.

The only ground upon which a provincial board may declare any municipal resolution,
ordinance, or order invalid is when such resolution, ordinance, or order is "beyond
the powers conferred upon the council or president making the same." Absolutely no
other ground is recognized by the law. A strictly legal question is before the
provincial board in its consideration of a municipal resolution, ordinance, or order.
The provincial (board's) disapproval of any resolution, ordinance, or order must be
premised specifically upon the fact that such resolution, ordinance, or order is
outside the scope of the legal powers conferred by law. If a provincial board passes
these limits, it usurps the legislative function of the municipal council or president.
Such has been the consistent course of executive authority. 20

Thus, the Sangguniang Panlalawigan was without the authority to disapprove Municipal
Resolution No. 43-89 for the Municipality of Bunawan clearly has the power to exercise the
right of eminent domain and its Sangguniang Bayan the capacity to promulgate said
resolution, pursuant to the earlier-quoted Section 9 of B.P. Blg. 337. Perforce, it follows that
Resolution No. 43-89 is valid and binding and could be used as lawful authority to petition
for the condemnation of petitioners' property.
PROVINCE OF CAMARINES SUR vs. CA
GR 103125

Facts:

On December 22, 1988, the Sangguniang Panlalawigan of the Province of Camarines Sur
passed a Resolution authorizing the Provincial Governor to purchase or expropriate property
contiguous to the provincial Capitol site, in order to establish a pilot farm for non-food and
non-traditional agricultural crops and a housing project for provincial government employees
Pursuant to the Resolution, the Province of Camarines Sur, through its Governor, filed two
separate cases for expropriation against Ernesto N. San Joaquin and Efren N. San Joaquin,
at the Regional Trial Court, Pili, Camarines Sur.
The San Joaquins moved to dismiss the complaints on the ground of inadequacy of the price
offered for their property. In an order, the trial court denied the motion to dismiss and
authorized the Province of Camarines Sur to take possession of the property upon the
deposit with the Clerk of Court the amount provisionally fixed by the trial court to answer
for damages that private respondents may suffer in the event that the expropriation cases
do not prosper.

The San Joaquins filed a motion for relief from the order, authorizing the Province of
Camarines Sur to take possession of their property and a motion to admit an amended
motion to dismiss. Both motions were denied in the order dated February 26, 1990.

In their petition before the Court of Appeals, the San Joaquins asked: (a) that Resolution of
the Sangguniang Panlalawigan be declared null and void; (b) that the complaints for
expropriation be dismissed; and (c) that the order denying the motion to dismiss and
allowing the Province of Camarines Sur to take possession of the property subject of the
expropriation and the order dated February 26, 1990, denying the motion to admit the
amended motion to dismiss, be set aside. They also asked that an order be issued to
restrain the trial court from enforcing the writ of possession, and thereafter to issue a writ
of injunction.

Asked by the Court of Appeals to give his Comment to the petition, the Solicitor General
stated that under Section 9 of the Local Government Code (B.P. Blg. 337), there was no
need for the approval by the Office of the President of the exercise by the Sangguniang
Panlalawigan of the right of eminent domain. However, the Solicitor General expressed the
view that the Province of Camarines Sur must first secure the approval of the Department of
Agrarian Reform of the plan to expropriate the lands of petitioners for use as a housing
project.
The Court of Appeals set aside the order of the trial court, allowing the Province of
Camarines Sur to take possession of private respondents' lands and the order denying the
admission of the amended motion to dismiss. It also ordered the trial court to suspend the
expropriation proceedings until after the Province of Camarines Sur shall have submitted the
requisite approval of the Department of Agrarian Reform to convert the classification of the
property of the private respondents from agricultural to non-agricultural land.

Issue: Whether the establishment of the Pilot Development Center and the housing project
are deemed for public use.
Ruling: Local government units have no inherent power of eminent domain and can
exercise it only when expressly authorized by the legislature. In delegating the power to
expropriate, the legislature may retain certain control or impose certain restraints on the
exercise thereof by the local governments.

While such delegated power may be a limited authority, it is complete within its limits.
Moreover, the limitations on the exercise of the delegated power must be clearly expressed,
either in the law conferring the power or in other legislations. It is the legislative branch of
the local government unit that shall determine whether the use of the property sought to be
expropriated shall be public, the same being an expression of legislative policy. The courts
defer to such legislative determination and will intervene only when a particular undertaking
has no real or substantial relation to the public use.

Statutes conferring the power of eminent domain to political subdivisions cannot be


broadened or constricted by implication. Section 9 of BP 337 does not intimate in the least
that local government units must first secure the approval of the Department of Land
Reform for the conversion of lands from agricultural to non-agricultural use, before they can
institute the necessary expropriation proceedings. Likewise, there is no provision in the
Comprehensive Agrarian Reform Law which expressly subjects the expropriation of
agricultural lands by local government units to the control of the Department of Agrarian
Reform. The rules on conversion of agricultural lands found in Section 4 (k) and 5 (1) of
Executive Order 129-A, Series of 1987, cannot be the source of the authority of the
Department of Agrarian Reform to determine the suitability of a parcel of agricultural land
for the purpose to which it would be devoted by the expropriating authority. While those
rules vest on the Department of Agrarian Reform the exclusive authority to approve or
disapprove conversions of agricultural lands for residential, commercial or industrial uses,
such authority is limited to the applications for reclassification submitted by the land owners
or tenant beneficiaries.

Further, there has been a shift from the literal to a broader interpretation of "public
purpose" or "public use" for which the power of eminent domain may be exercised. The old
concept was that the condemned property must actually be used by the general public (e.g.
roads, bridges, public plazas, etc.) before the taking thereof could satisfy the constitutional
requirement of "public use".

Under the new concept, "public use" means public advantage, convenience or benefit, which
tends to contribute to the general welfare and the prosperity of the whole community, like a
resort complex for tourists or housing project. The expropriation of the property authorized
by Resolution 129, Series of 1988, is for a public purpose. The establishment of a pilot
development center would inure to the direct benefit and advantage of the people of the
Province of Camarines Sur. Once operational, the center would make available to the
community invaluable information and technology on agriculture, fishery and the cottage
industry. Ultimately, the livelihood of the farmers, fishermen and craftsmen would be
enhanced. The housing project also satisfies the public purpose requirement of the
Constitution. Housing is a basic human need. Shortage in housing is a matter of state
concern since it directly and significantly affects public health, safety, the environment and
in sum the general welfare. Thus, the decision of the Court of Appeals is set aside insofar as
it (a) nullifies the trial court's order allowing the Province of Camarines Sur to take
possession of the property of the San Joaquins; (b) orders the trial court to suspend the
expropriation proceedings; and (c) requires the Province of Camarines Sur to obtain the
approval of the Department of Agrarian Reform to convert or reclassify the property of the
San Joaquins property from agricultural to non-agricultural use.

BARANGAY SAN ROQUE vs. HEIRS OF PASTOR

Facts:
1 Barangay San Roque filed an expropriation suit with the MTC against the properties of
the respondents.
a MTC: dismissed it for lack of jurisdiction. It ruled that an action for eminent domain
is incapable of pecuniary estimation, hence, jurisdiction lies with the RTC.

2 The case was then filed to the RTC.


a RTC: dismissed the same for lack of jurisdiction. RTC ruled that an action for
eminent domain involved title to real property and the value of the said property is
determinative of jurisdiction. Since the value of the property involved in this case is
1,740.00 (less than 20,000), jurisdiction is with the MTC.

Issued/Held: Which court has jurisdiction over actions for eminent domain? RTC.

Ratio: Actions for eminent domain are incapable of pecuniary estimation, and thus,
jurisdiction lies with the RTC.

In determining whether an action is one which subject matter of which is not capable of
pecuniary estimation, the criterion is of first ascertaining the nature of the principal action
or remedy sought. If it is primarily for the recovery of a sum of money, the claim is
considered capable of pecuniary estimation.

However, where the basic issue is something other than the right to recover a sum of
money, or where the money claim is purely incidental to, or a consequence of, the principal
relief sought, such actions are cases where the subject of the litigation may not be
estimated in terms of money, and are cognizable exclusively by the RTC.

The rationale for the above rule is that the second class of cases (cases not capable of
pecuniary estimation), besides the determination of damages, demands an inquiry into
other factors which the law deemed to be more within the competence of the RTC.

Action for eminent domain involve 2 phases:

1 First Phase - concerned with the determination of:


a Authority of the plaintiff to exercise the power of eminent domain
b Propriety of its exercise in the context of the facts involved in the suit
It ends with an order of condemnation (if not of a dismissal of the action)
declaring that the plaintiff has a lawful right to take the property sought to be
condemned, for the public use or purpose described in the complaint, upon the
payment of just compensation to be determined as of the date of the filing of the
complaint.

2 Second Phase concerned with the determination by the court of the just
compensation for the property sought to be taken

This is done by the court with the assistance of 3 commissioners. The order fixing the
just compensation on the basis of the evidence before, and findings of, the
commissioners would be final too. It would finally dispose of the 2 nd stage of the suit,
and leave nothing more to be done by the court regarding the issue

It should be stressed that the primary consideration in an expropriation suit is whether the
government or any of its instrumentalities has complied with the requisites for the taking of
private property. Hence, the courts shall determine: (1) the authority of the government
entity; (2) the necessity of the expropriation; (3) observance of due process.

In the case at bar, the subject of an expropriation suit is the governments exercise of
eminent domain, a matter that is incapable of pecuniary estimation.

Although the value of the property to be expropriated is estimated in monetary terms. This
is mere incidental to the expropriation suit. Indeed, the amount is determined only after the
court is satisfied with the propriety of the expropriation.
MUNICIPALITY OF PARANAQUE vs. VM REALTY
GR. No. 127820, July 20, 1998

Facts:

Pursuant to Sangguniang Bayan Resolution No. 93-95, Series of 1993, the


Municipality of Paranaque filed on September 20, 1993, a Complaint for expropriation
against V.M. Realty Corporation over two parcels of land, with a combined area of about
10,000 square meters, located at Wakas, San Dionisio, Paranaque, Metro Manila. Allegedly,
the complaint was filed for the purpose of alleviating the living conditions of the
underprivileged by providing homes for the homeless through a socialized housing project.
Parenthetically, it was also for this stated purpose that petitioner, pursuant to
its Sangguniang Bayan Resolution No. 577, Series of 1991, previously made an offer to
enter into a negotiated sale of the property with private respondent, which the latter did not
accept

Subsequently petitioners filed a complaint in the RTC. Acting on petitioners motion, the
court issued an Order dated February 4, 1994, authorizing petitioner to take possession of
the subject property upon deposit with its clerk of court of an amount equivalent to 15
percent of its fair market value based on its current tax declaration.

On February 21, 1994, VM realty filed its Answer containing affirmative defenses and a
counterclaim, alleging in the main that (a) the complaint failed to state a cause of action
because it was filed pursuant to a resolution and not to an ordinance as required by RA
7160 (the Local Government Code. RTC and CA ruled in favor of VM Realty.

Issue:

W/N a resolution duly approved by the municipal council has the same force and
effect of an ordinance and will not deprive an expropriation case of a valid cause of action.

Ruling:
NO. he power of eminent domain is lodged in the legislative branch of government,
which may delegate the exercise thereof to LGUs, other public entities and public utilities.
[25]
An LGU may therefore exercise the power to expropriate private property only when
authorized by Congress and subject to the latters control and restraints, imposed through
the law conferring the power or in other legislations. [26] In this case, Section 19 of RA 7160,
which delegates to LGUs the power of eminent domain, also lays down the parameters for
its exercise. It provides as follows:

Section 19. Eminent Domain. A local government unit may, through its
chief executive and acting pursuant to an ordinance, exercise the power of
eminent domain for public use, or purpose, or welfare for the benefit of the
poor and the landless, upon payment of just compensation, pursuant to the
provisions of the Constitution and pertinent laws: Provided, however, That
the power of eminent domain may not be exercised unless a valid and
definite offer has been previously made to the owner, and such offer was
not accepted: Provided, further, That the local government unit may
immediately take possession of the property upon the filing of the
expropriation proceedings and upon making a deposit with the proper court
of at least fifteen percent (15%) of the fair market value of the property
based on the current tax declaration of the property to be
expropriated: Provided, finally, That, the amount to be paid for the
expropriated property shall be determined by the proper court, based on
the fair market value at the time of the taking of the property. (Emphasis
supplied)

Thus, the following essential requisites must concur before an LGU can
exercise the power of eminent domain:

1. An ordinance is enacted by the local legislative council authorizing the local


chief executive, in behalf of the LGU, to exercise the power of eminent
domain or pursue expropriation proceedings over a particular private
property.

2. The power of eminent domain is exercised for public use, purpose or


welfare, or for the benefit of the poor and the landless.

3. There is payment of just compensation, as required under Section 9, Article


III of the Constitution, and other pertinent laws.

4. A valid and definite offer has been previously made to the owner of the
property sought to be expropriated, but said offer was not accepted.

In the case at bar, the local chief executive sought to exercise the power of eminent
domain pursuant to a resolution of the municipal council. Thus, there was no compliance
with the first requisite that the mayor be authorized through an ordinance. Petitioner
cites Camarines Sur vs. Court of Appeals to show that a resolution may suffice to support
the exercise of eminent domain by an LGU. This case, however, is not in point because the
applicable law at that time was BP 337, the previous Local Government Code, which had
provided that a mere resolution would enable an LGU to exercise eminent domain. In
contrast, RA 7160, the present Local Government Code which was already in force when the
Complaint for expropriation was filed, explicitly required an ordinance for this purpose.
We are not convinced by petitioners insistence that the terms resolution and ordinance
are synonymous. A municipal ordinance is different from a resolution. An ordinance is a law,
but a resolution is merely a declaration of the sentiment or opinion of a lawmaking body on
a specific matter. An ordinance possesses a general and permanent character, but a
resolution is temporary in nature. Additionally, the two are enacted differently -- a third
reading is necessary for an ordinance, but not for a resolution, unless decided otherwise by
a majority of all theSanggunian members.

If Congress intended to allow LGUs to exercise eminent domain through a mere


resolution, it would have simply adopted the language of the previous Local Government
Code. But Congress did not. In a clear divergence from the previous Local Government
Code, Section 19 of RA 7160 categorically requires that the local chief executive act
pursuant to an ordinance.Indeed, [l]egislative intent is determined principally from the
language of a statute. Where the language of a statute is clear and unambiguous, the law is
applied according to its express terms, and interpretation would be resorted to only where a
literal interpretation would be either impossible or absurd or would lead to an injustice. In
the instant case, there is no reason to depart from this rule, since the law requiring an
ordinance is not at all impossible, absurd, or unjust.

CITY OF CEBU vs. COURT OF APPEALS


G.R. No. 109173. July 5, 1996

Facts:

Merlita Cardeno is the owner of a parcel of land with an area of 2,019 square meters located
at Sitio Sto. Nino, Alaska-Mambaling. On February 25,1992, City of Cebu filed a complaint
for eminent domain against Cardeno with RTC of Cebu City seeking to expropriate the said
parcel of land. The complaint was initiated pursuant to Resolution No. 404 and Ordinance
No.1418, dated February 17,1992, of the Sangguniang Panlungsod of Cebu City authorizing
the City Mayor to expropriate the said parcel of land for the purpose of providing a
socialized housing project for the landless and low-income city residents.

Cardeno filed a motion to dismiss the said complaint on the ground that Cebu City failed to
comply with one of the conditions precedent to the exercise of the power of eminent domain
by a local government unit for there has been no valid and definite offer previously made.
Cebu City sought to establish compliance with the abovecited requirement by alleging in its
Comment and Opposition the following:

7. To further pursue its desire to acquire the property concerned, the plaintiff
made on October 28,1991, another offer to Mrs. Cardeno, through her lawyer,
Atty. Omar Redula, for the purchase of her property in the amount of Four
Hundred Seventy Eight Thousand (P478,000.00) Pesos. x x x;
8. The said offer was again refused, thus the resort by the plaintiff to
expropriation.

The RTC nevertheless dismissed the complaint and ruled that the import or meaning of the
allegations in the complaint does not convey or connote the same meaning or import or
even approximate, the condition precedent required that unless a valid and definite offer
has been previously made to the owner. Aggrieved, Cebu City filed a special civil action
for certiorari before the Court of Appeals which, however, affirmed the above ruling of the
RTC.

Argument: Cebu City reiterates that the complaint sufficiently states compliance with the
requirement of a valid and definite offer, while Cardeno insists that the term negotiations is
too broad to be equated with the said requirement. Elaborating, Cardeno posited that by
definition, negotiations run the whole range of acts preparatory to concluding an agreement,
from the preliminary correspondence; the fixing of the terms of the agreement; the price;
the mode of payment; obligations of the parties may conceive as necessary to their
agreement. Thus, negotiations by itself may pertain to any of the foregoing and does not
automatically mean the making of a valid and definite offer.

Issue:

Whether or not Cebu City can proceed with the expropriation of the land?

Ruling:

Yes. A complaint should not be dismissed for insufficiency unless it appears clearly from the
face of the complaint that the plaintiff is not entitled to any relief under any state of facts
which could be proved within the facts alleged therein. Likewise, in Marcopper Mining
Corporation v. Garcia, we sanctioned the act of the trial court in considering, in addition to
the complaint, other pleadings submitted by the parties in deciding whether or not the
complaint should be dismissed for lack of cause of action. The same liberality should be
applied in the instant case where an examination of the Comment and Opposition leaves no
room for doubt that Cebu City had indeed made a valid and definite offer to Cardeno as
required by law.

Furthermore, a closer scrutiny reveals that even on the face of the complaint alone, there is
extant a cause of action. Cebu City avers in paragraph I thereof that, "x x x. Under R.A.
7160, Sec. 9 thereof, the City of Cebu is legally vested with the power of eminent domain
and pursuant thereto is filing this petition/complaint as authorized by Ordinance No.1418
passed by the Sangguniang Panlungsod on February 17,1992, a photocopy of which is
herein attached as Annex A, and made an integral part of this complaint. x x

All documents attached to a complaint, the due execution and genuineness of which are not
denied under oath by the defendant, must be considered as part of the complaint without
need of introducing evidence thereon. Additionally, the general rule is that a motion to
dismiss hypothetically admits the truth of the facts alleged in the complaint. Thus,
Ordinance No.1418, with all its provisions, is not only incorporated into the complaint for
eminent domain filed by Cebu City, but is also deemed admitted by Cardeno. A perusal of
the copy of said ordinance which has been annexed to the complaint shows the fact that
Cebu City had made a previous valid and definite offer to Cardeno.

And as a fitting finale to this controversy, the principle enunciated in both


the Tan and Marcopper cases is here reiterated: The rules of procedure are not to be applied
in a very rigid, technical sense; rules of procedure are used only to help secure substantial
justice. If a technical and rigid enforcement of the rules is made their aim would be
defeated. Where the rules are merely secondary in importance are made to override the
ends of justice; the technical rules had been misapplied to the prejudice of the substantial
right of a party, said rigid application cannot be countenanced.
The aforequoted doctrine finds compelling application in the case at bench. Nothing else
was acomplished by the dismissal of the complaint for eminent domain but a considerable
delay in the proceedings. The dismissal of the complaint did not bar Cebu City from filing
another eminent domain case and from correcting its alleged error by the mere expedient of
changing paragraph VII thereof. Indeed, precious time has been wasted while the salutary
objectives of Ordinance No. 1418 of the City of Cebu have been put on hold by a quarrel
over technical matters.

HEIRS OF JUANCHO ARDONA vs. REYES


Gr Nos. L-60549, 60553 to 60555 October 26, 1983

Facts:

The Philippine Tourism Authority (PTA) filed 4 complaints with the Court of First Instance of
Cebu City for the expropriation of 282 hectares of rolling land situated in barangay Alubog
and Babag, Cebu City, under PTAs express authority to acquire by purchase, by negotiation
or by condemnation proceedings any private land within and without the tourist zones for
the development into integrated resort complexes of selected and well-defined geographic
areas with potential tourism value. (for tourism purposes). The heirs of Juancho Ardona et.
Al, filed their oppositions, alleging that the taking is not for public use; that there is no
specific constitutional provision authorizing the taking of private property for tourism
purposes; that assuming that PTA has such power, the intended use cannot be paramount to
the determination of the land as a land reform area; that limiting the amount of
compensation by Legislative fiat is constitutionally repugnant; and that since the land is
under the land reform program, it is the Court of Agrarian Relations and not the Court of
First Instance that has jurisdiction over the expropriation cases.

The issues raised by the petitioners revolve around the proposition that the actions to
expropriate their properties are constitutionally infirm because nowhere in the Constitution
allows the taking of private property for the promotion of tourism.
Issue:

Whether the expropriation of parcels of land for the purpose of constructing a sports
complex by the Philippine Tourism Authority be considered taking for public use.

Ruling:
The States power of eminent domain extends to the expropriation of land for tourism
purposes although this specific objective is not expressed in the Constitution.The
petitioners look for the word tourism in the Constitution. Understandably the search would
be in vain. The policy objectives of the framers can be expressed only in general terms such
as social justice, local autonomy, conservation and development of the national patrimony,
public interest, and general welfare, among others. The programs to achieve these
objectives vary from time to time and according to place. To freeze specific programs like
tourism into express constitutional provisions would make the Constitution more prolix than
a bulky code and require of the framers prescience beyond Delphic proportions. The
particular mention in the Constitution of agrarian reform and the transfer of utilities and
other private enterprises to public ownership merely underscores the magnitude of the
problems sought to be remedied by these programs. They do not preclude nor limit the
exercise of the power of eminent domain for such purposes like tourism and other
development programs.

FILSTREAM INTERNATIONAL INC. vs. CA


GR 125218 and GR 128077. 23 January 1998
Facts:
Filstream International, Inc., is the registered owner of the properties consisting of adjacent
parcels of land situated in Antonio Rivera Street, Tondo II, Manila, with a total area of
3,571.10 square meters (TCT 203937, 203936, 169198, 169199, 169200 and 169202 of
the Register of Deeds of Manila).
On 7 January 1993, Filstream filed an ejectment suit before the MTC of Manila against the
occupants of the parcels of land on the grounds of termination of the lease contract and
non-payment of rentals.
Judgment was rendered by the MTC on September 14, 1993 ordering private respondents to
vacate the premises and pay back rentals to Filstream. Not satisfied, malit, et. al. appealed
the decision to the RTC of Manila which in turn affirmed the decision of the MTC. Still not
content, Malit, et. al. proceeded to the Court of Appeals via a petition for review.
The result however remained the same as the appellate court affirmed the decision of the
RTC in its decision dated 25 August 1994. Thereafter, no further action was taken by Malit,
et. al., as a result of which the decision in the ejectment suit became final and executory.
However, during the pendency of the ejectment proceedings Malit, et. al. filed on 25 May
1993, a complaint for Annulment of Deed of Exchange against Filstream before the RTC of
Manila.
It was at this stage that City of Manila came into the picture when the city government
approved Ordinance 7813 on 5 November 1993, authorizing Mayor Alfredo S. Lim to initiate
the acquisition by negotiation, expropriation, purchase, or other legal means certain parcels
of land which formed part of Filstream's properties then occupied by Malit, et. al.
Subsequently, the City of Manila approved Ordinance 7855 declaring the expropriation of
certain parcels of land situated along Antonio Rivera and Fernando Ma. Guerrero streets in
Tondo, Manila which were owned by Mr. Enrique Quijano Gutierrez, Filstream's predecessor-
in-interest.
The said properties were to be sold and distributed to qualified tenants of the area pursuant
to the Land Use Development Program of the City of Manila. On 23 May 1994, the City of
Manila filed a complaint for eminent domain before the RTC of Manila, seeking to
expropriate the parcels of land owned by Filstream which are situated at Antonio Rivera
Street, Tondo II, Manila.
Pursuant to the complaint filed by the City of Manila, the trial court issued a Writ of
Possession in favor of the former which ordered the transfer of possession over the disputed
premises to the City of Manila.
Filstream filed a motion to dismiss the complaint for eminent domain as well as a motion to
quash the writ of possession. On 30 September 1994, the RTC of Manila issued an order
denying Filstream's motion to dismiss and the motion to quash the Writ of Possession.
Filstream filed a motion for reconsideration as well as a supplemental motion for
reconsideration seeking the reversal of the order but the same were denied.
Still, Filstream filed a subsequent motion to be allowed to file a second motion for
reconsideration but it was also denied.
Aggrieved, Filstream filed on 31 March 1996, a Petition for Certiorari with the Court of
Appeals (CAGR SP 36904) seeking to set aside the RTC order. On 18 March 1996, the
appellate court dismissed the petition.
Filsteream filed a motion for reconsideration and attached clearer copies of the pertinent
documents and papers pursuant to Section 2(a), Rule 6 of the Revised Internal Rules of the
Court of Appeals. But on 20 May 1996, the appellate court issued a resolution denying the
motion as petitioner failed to submit clearer and readable copies of the pleadings. This
prompted Filstream to proceed to the Supreme Court by filing a petition for review on
certiorari.
Meanwhile, owing to the finality of the decision in the ejectment suit, the MTC of Manila,
Branch 15, upon motion of Filstream, issued a Writ of Execution as well as a Notice to
Vacate the disputed premises. Malit, et. al. filed a Motion to Recall/Quash the Writ of
Execution and Notice to Vacate alleging the existence of a supervening event in that the
properties subject of the dispute have already been ordered condemned in an expropriation
proceeding in favor of the City of Manila for the benefit of the qualified occupants thereof,
thus execution shall be stayed.
For its part, the City of Manila filed on 13 March 1996, a motion for intervention with prayer
to stay/quash the writ of execution on the ground that it is the present possessor of the
property subject of execution. In its order dated 14 March 1996, the MTC of Manila denied
Malit, et. al.'s motion as it found the allegations therein bereft of merit and upheld the
issuance of the Writ of Execution and Notice to Vacate in Filstream's favor.
Subsequently, the trial court also denied the motion filed by the City of Manila. On 22 April
1996, the trial court issued an order commanding the demolition of the structure erected on
the disputed premises. To avert the demolition, Malit, et. al. filed before the RTC of Manila,
(Branch 14, Civil Case 96-78098) a Petition for Certiorari and Prohibition with prayer for the
issuance of a temporary restraining order and preliminary injunction . On 15 May 1996, the
City of Manila filed its Petition for Certiorari and Prohibition with prayer for the issuance of a
temporary restraining order and preliminary injunction which was raffled to Branch 23 of the
RTC of Manila (Civil Case 96-78382), seeking the reversal of the orders issued by the MTC
of Manila, Branch 14.
Thereafter, upon motion filed by the City of Manila, an order was issued by the RTC of
Manila, Branch 10, ordering the consolidation of Civil Case 96-78382 with Civil Case 96-
78098 pending before Branch 14 of the RTC of Manila. Injunctions were issued. Filstream
then filed a motion for reconsideration from the order of denial but pending resolution of
this motion, it filed a motion for voluntary inhibition of the presiding judge of the RTC of
Manila, Branch 14. The motion for inhibition was granted 25 and as a result, the
consolidated cases (Civil Cases 96-78382 and 96-78098) were re-raffled to the RTC of
Manila, Branch 33. During the proceedings before the RTC of Manila, Branch 33, Filstream
moved for the dismissal of the consolidated cases for forum shopping because the same
parties, causes of action and subject matter involved therein have already been disposed of
in the decision in the ejectment case (Civil Case 140817) which has already become final
and executory prior to the filing of these consolidated cases.
On 9 December 1996, the RTC of Manila, Branch 33 ordered the dismissal of Civil Cases 96-
78382 and 96-78098 due to forum shopping. Immediately thereafter, Filstream filed an Ex-
parte Motion for Issuance of an Alias Writ of Demolition and Ejectment and a supplemental
motion to the same dated January 10 and 13, 1997, respectively, before the MTC of Manila,
Branch 15, which promulgated the decision in the ejectment suit.
On January 1997, the court granted the motion and issued the corresponding writ of
demolition. As a consequence of the dismissal of the consolidated cases, Malit, et. al. filed a
Petition for Certiorari and Prohibition with prayer for the issuance of a temporary restraining
order and preliminary injunction before the Court of Appeals (CA-GR SP 43101). At the
conclusion of the hearing for the issuance of a writ of preliminary injunction, the Court of
Appeals, in its resolution dated 18 February 1997, found merit in Malit, et. al.'s allegations
in support of their application of the issuance of the writ and granted the same. Filstream
filed a Petition for Certiorari under Rule 65.
Issue:
Whether there is violation of due process against Filstream in the manner its properties
were expropriated and condemned in favor of the City of Manila.

Held:
That only a few could actually benefit from the expropriation of the property does not
diminish its public use character. It is simply not possible to provide all at once land and
shelter for all who need them. Corollary to the expanded notion of public use, expropriation
is not anymore confined to vast tracts of land and landed estates. It is therefore of no
moment that the land sought to be expropriated in this case is less than half a hectare only.
Through the years, the public use requirement in eminent domain has evolved into a flexible
concept, influenced by changing conditions. Public use now includes the broader notion of
indirect public benefit or advantage, including in particular, urban land reform and housing.
The Court takes judicial notice of the fact that urban land reform has become a paramount
task in view of the acute shortage of decent housing in urban areas particularly in Metro
Manila. Nevertheless, despite the existence of a serious dilemma, local government units
are not given an unbridled authority when exercising their power of eminent domain in
pursuit of solutions to these problems.
The basic rules still have to be followed, which are as follows: "no person shall be deprived
of life, liberty, or property without due process of law, nor shall any person be denied the
equal protection of the laws; private property shall not be taken for public use without just
compensation". Thus, the exercise by local government units of the power of eminent
domain is not without limitations. Even Section 19 of the 1991 Local Government Code is
very explicit that it must comply with the provisions of the Constitution and pertinent laws.
Very clear from Sections 9 and 10 of Republic Act 7279 (Urban Development and Housing
Act of 1992) are the limitations with respect to the order of priority in acquiring private
lands and in resorting to expropriation proceedings as a means to acquire the same. Private
lands rank last in the order of priority for purposes of socialized housing.
In the same vein, expropriation proceedings are to be resorted to only when the other
modes of acquisition have been exhausted. Compliance with these conditions must be
deemed mandatory because these are the only safeguards in securing the right of owners of
private property to due process when their property is expropriated for public use. There is
nothing in the records that would indicate that City of Manila complied with Section 9 and
Section 10 of RA 7279. Filstream's properties were expropriated and ordered condemned in
favor of the City of Manila sans any showing that resort to the acquisition of other lands
listed under Section 9 of RA 7279 have proved futile. Evidently, there was a violation of
Filstream's right to due process which must accordingly be rectified.

HAGONOY MARKET VENDORS ASSOCIATION vs. MUNICIPALITY OF HAGONOY

Facts: On October 1, 1996, the Sangguniang Bayan of Hagonoy, Bulacan, enacted an


ordinance, Kautusan Blg. 28, which increased the stall rentals of the market vendors in
Hagonoy. Article 3 provided that it shall take effect upon approval. The subject ordinance
was posted from November 4-25, 1996. In the last week of November, 1997, the
petitioners members were personally given copies of the approved Ordinance and were
informed that it shall be enforced in January, 1998. On December 8, 1997, the petitioners
President filed an appeal with the Secretary of Justice assailing the constitutionality of the
tax ordinance. Petitioner claimed it was unaware of the posting of the ordinance.
Respondent opposed the appeal. It contended that the ordinance took effect on October 6,
1996 and that the ordinance, as approved, was posted as required by law. Hence, it was
pointed out that petitioners appeal, made over a year later, was already time-barred.
The Secretary of Justice dismissed the appeal on the ground that it was filed out of
time, i.e., beyond 30 days from the effectivity of the Ordinance on October 1, 1996, as
prescribed under Section 187 of the 1991 LGC. Citing the case of Taada vs. Tuvera, the
Secretary of Justice held that the date of effectivity of the subject ordinance retroacted to
the date of its approval in October 1996, after the required publication or posting has been
complied with, pursuant to Section 3 of said ordinance. After its motion for reconsideration
was denied, petitioner appealed to the Court of Appeals. Petitioner did not assail the finding
of the Secretary of Justice that their appeal was filed beyond the reglementary period.
Instead, it urged that the Secretary of Justice should have overlooked this mere
technicality and ruled on its petition on the merits.
Unfortunately, its petition for review was dismissed by the Court of Appeals for being
formally deficient as it was not accompanied by certified true copies of the assailed
Resolutions of the Secretary of Justice. Undaunted, the petitioner moved for reconsideration
but it was denied.

Issues:

1. W/N the CA was correct in dismissing the petition for review for petitioners failure to
attach certified true copies of the assailed Resolutions of the Secretary of Justice. YES.

2. W/N the appeal was time-barred. YES.

3. WON the period to appeal should be counted not from the time the ordinance took effect
in 1996 but from the time its members were personally given copies of the approved
ordinance in November 1997. NO

Ruling:

1. In its Motion for Reconsideration before the Court of Appeals, the petitioner satisfactorily
explained the circumstances relative to its failure to attach to its appeal certified true copies
of the assailed Resolutions of the Secretary of Justice, thus: during the preparation of the
petition on October 21, 1998, it was raining very hard due to (t)yphoon Loleng. When the
petition was completed, copy was served on the Department of Justice at about (sic) past
4:00 p.m. of October 21, 1998, with (the) instruction to have the Resolutions of the
Department of Justice be stamped as certified true copies. However, due to bad weather,
the person in charge (at the Department of Justice) was no longer available to certify to
(sic) the Resolutions. The following day, October 22, 1998, was declared a non-working
holiday because of (t)yphoon Loleng. Thus, petitioner was again unable to have the
Resolutions of the Department of Justice stamped certified true copies. In the morning of
October 23, 1998, due to time constraint(s) , herein counsel served a copy by personal
service on (r)espondents lawyer at (sic) Malolos, Bulacan, despite the flooded roads and
heavy rains. However, as the herein counsel went back to Manila, (official business in)
government offices were suspended in the afternoon and the personnel of the Department
of Justice tasked with issuing or stamping certified true copies of their Resolutions were no
longer available. To avoid being time-barred in the filing of the (p)etition, the same was
filed with the Court of Appeals as is.

CA erred in dismissing petitioners appeal on the ground that it was formally deficient. It is
clear from the records that the petitioner exerted due diligence to get the copies of its
appealed Resolutions certified by the Department of Justice, but failed to do so on account
of typhoon Loleng. Under the circumstances, respondent appellate court should have
tempered its strict application of procedural rules in view of the fortuitous event considering
that litigation is not a game of technicalities

2. The applicable law is Section 187 of the 1991 LGC which provides: SEC. 187. Procedure
for Approval and Effectivity of Tax Ordinances and Revenue Measures; Mandatory Public
Hearings. - The procedure for the approval of local tax ordinances and revenue measures
shall be in accordance with the provisions of this Code: Provided, That public hearings shall
be conducted for the purpose prior to the enactment thereof: Provided, further, That any
question on the constitutionality or legality of tax ordinances or revenue measures may be
raised on appeal within thirty (30) days from the effectivity thereof to the Secretary of
Justice who shall render a decision within sixty (60) days from the receipt of the appeal:
Provided, however, That such appeal shall not have the effect of suspending the effectivity
of the ordinance and accrual and payment of the tax, fee or charge levied therein:
Provided, finally, That within thirty (30) days after receipt of the decision or the lapse of the
sixty-day period without the Secretary of Justice acting upon the appeal, the aggrieved
party may file appropriate proceedings.

The aforecited law requires that an appeal of a tax ordinance or revenue measure should be
made to the Secretary of Justice within thirty (30) days from effectivity of the ordinance and
even during its pendency, the effectivity of the assailed ordinance shall not be suspended.
In the case at bar, Municipal Ordinance No. 28 took effect in October 1996. Petitioner filed
its appeal only in December 1997, more than a year after the effectivity of the ordinance in
1996. Clearly, the Secretary of Justice correctly dismissed it for being time-barred. At this
point, it is apropos to state that the timeframe fixed by law for parties to avail of their legal
remedies before competent courts is not a mere technicality that can be easily brushed
aside. The periods stated in Section 187 of the LGC are mandatory. Ordinance No. 28 is a
revenue measure adopted by the municipality of Hagonoy to fix and collect public market
stall rentals. Being its lifeblood, collection of revenues by the government is of paramount
importance. The funds for the operation of its agencies and provision of basic services to its
inhabitants are largely derived from its revenues and collections. Thus, it is essential that
the validity of revenue measures is not left uncertain for a considerable length of time.
Hence, the law provided a time limit for an aggrieved party to assail the legality of revenue
measures and tax ordinances.

3. Petitioner insists that it was unaware of the approval and effectivity of the subject
ordinance in 1996 on two (2) grounds: first, no public hearing was conducted prior to the
passage of the ordinance and, second, the approved ordinance was not posted.

Petitioners bold assertion that there was no public hearing conducted prior to the passage
of Kautusan Blg. 28 is belied by its own evidence. In petitioners two (2) communications
with the Secretary of Justice, it enumerated the various objections raised by its members
before the passage of the ordinance in several meetings called by the Sanggunian for the
purpose. These show beyond doubt that petitioner was aware of the proposed increase and
in fact participated in the public hearings therefor. The respondent municipality likewise
submitted the Minutes and Report of the public hearings conducted by the Sangguniang
Bayans Committee on Appropriations and Market on February 6, July 15 and August 19, all
in 1996, for the proposed increase in the stall rentals.

Petitioner cannot gripe that there was practically no public hearing conducted as its
objections to the proposed measure were not considered by the Sangguniang Bayan. To be
sure, public hearings are conducted by legislative bodies to allow interested parties to
ventilate their views on a proposed law or ordinance. These views, however, are not
binding on the legislative body and it is not compelled by law to adopt the same.
Sanggunian members are elected by the people to make laws that will promote the general
interest of their constituents. They are mandated to use their discretion and best judgment
in serving the people. Parties who participate in public hearings to give their opinions on a
proposed ordinance should not expect that their views would be patronized by their
lawmakers.

On the issue of publication or posting, Section 188 of the LGC provides: Section 188.
Publication of Tax Ordinance and Revenue Measures. Within ten (10) days after their
approval, certified true copies of all provincial, city, and municipal tax ordinances or revenue
measures shall be published in full for three (3) consecutive days in a newspaper of local
circulation; Provided, however, That in provinces, cities and municipalities where there are
no newspapers of local circulation, the same may be posted in at least two (2) conspicuous
and publicly accessible places.
Sangguniang Bayan of the Municipality of Hagonoy, Bulacan, presented evidence which
clearly shows that the procedure for the enactment of the assailed ordinance was complied
with. Municipal Ordinance No. 28 was enacted by the Sangguniang Bayan of Hagonoy on
October 1, 1996. Then Acting Municipal Mayor Maria Garcia Santos approved the
Ordinance on October 7, 1996. After its approval, copies of the Ordinance were given to the
Municipal Treasurer on the same day. On November 9, 1996, the Ordinance was approved
by the Sangguniang Panlalawigan. The Ordinance was posted during the period from
November 4 - 25, 1996 in three (3) public places, viz: in front of the municipal building, at
the bulletin board of the Sta. Ana Parish Church and on the front door of the Office of the
Market Master in the public market. Posting was validly made in lieu of publication as there
was no newspaper of local circulation in the municipality of Hagonoy. This fact was known
to and admitted by petitioner. Thus, petitioners ambiguous and unsupported claim that it
was only sometime in November 1997 that the Provincial Board approved Municipal
Ordinance No. 28 and so the posting could not have been made in November 1996 was
sufficiently disproved by the positive evidence of respondent municipality. Given the
foregoing circumstances, petitioner cannot validly claim lack of knowledge of the approved
ordinance. The filing of its appeal a year after the effectivity of the subject ordinance is fatal
to its cause.

**Even on the substantive points raised, the petition must fail. Section 6c.04 of the 1993
Municipal Revenue Code and Section 191 of the LGC limiting the percentage of increase that
can be imposed apply to tax rates, not rentals. Neither can it be said that the rates were
not uniformly imposed or that the public markets included in the Ordinance were
unreasonably determined or classified. To be sure, the Ordinance covered the three (3)
concrete public markets: the two-storey Bagong Palengke, the burnt but reconstructed
Lumang Palengke and the more recent Lumang Palengke with wet market. However, the
Palengkeng Bagong Munisipyo or Gabaldon was excluded from the increase in rentals as it is
only a makeshift, dilapidated place, with no doors or protection for security, intended for
transient peddlers who used to sell their goods along the sidewalk.
REPUBLIC vs. CA
G.R. No. 146587 July 2, 2002

FACTS:

Petitioner (PIA) instituted expropriation proceedings covering a total of 544,980 square


meters of contiguous land situated along MacArthur Highway, Malolos, Bulacan, to be
utilized for the continued broadcast operation and use of radio transmitter facilities for the
Voice of the Philippines project.

Petitioner made a deposit of P517,558.80, the sum provisionally fixed as being the
reasonable value of the property. On 26 February 1979, or more than 9 years after the
institution of the expropriation proceedings, the trial court issued this order condemning the
property and ordering the plaintiff to pay the defendants the just compensation for the
property.

It would appear that the National Government failed to pay the respondents the just
compensation pursuant to the foregoing decision. The respondents then filed a
manifestation with a motion seeking payment for the expropriated property. In response,
the court issued a writ of execution for the implementation thereof.
Meanwhile, Pres. Estrada issued Proc. No. 22 transferring 20 hectares of the expropriated
land to the Bulacan State University.

Despite the courts order, the Santos heirs remained unpaid and no action was on their case
until petitioner filed its manifestation and motion to permit the deposit in court of the
amount P4,664,000 by way of just compensation.

The Santos heirs submitted a counter-motion to adjust the compensation from P6/sq.m. as
previously fixed to its current zonal value of P5,000/sq.m. or to cause the return of the
expropriated property.

The RTC Bulacan ruled in favor of the Santos heirs declaring its 26 February 1979 Decision
to be unenforceable on the ground of prescription in accordance with Sec. 6, Rule 39 of the
1964/1997 ROC which states that a final and executory judgment or order may be executed
on motion within 5 years from the date of its entry. RTC denied petitioners Motion to Permit
Deposit and ordered the return of the expropriated property to the heirs of Santos.

ISSUES:

1. WON the petitioner may appropriate the property


2. WON the respondents are entitled to the return of the property in question

HELD:

1. The right of eminent domain is usually understood to be an ultimate right of the


sovereign power to appropriate any property within its territorial sovereignty for a public
purpose. Fundamental to the independent existence of a State, it requires no recognition by
the Constitution, whose provisions are taken as being merely confirmatory of its presence
and as being regulatory, at most, in the due exercise of the power. In the hands of the
legislature, the power is inherent, its scope matching that of taxation, even that of police
power itself, in many respects. It reaches to every form of property the State needs for
public use and, as an old case so puts it, all separate interests of individuals in property are
held under a tacit agreement or implied reservation vesting upon the sovereign the right to
resume the possession of the property whenever the public interest so requires it.

The ubiquitous character of eminent domain is manifest in the nature of the expropriation
proceedings. Expropriation proceedings are not adversarial in the conventional sense, for
the condemning authority is not required to assert any conflicting interest in the property.
Thus, by filing the action, the condemnor in effect merely serves notice that it is taking title
and possession of the property, and the defendant asserts title or interest in the property ,
not to prove a right to possession, but to prove a right to compensation for the taking.

Obviously, however, the power is not without its limits: first, the taking must be for public
use, and second, that just compensation must be given to the private owner of the property.
These twin proscriptions have their origin in the recognition of the necessity for achieving
balance between the State interests, on the one hand, and private rights, upon the other
hand, by effectively restraining the former and affording protection to the latter. In
determining public use, two approaches are utilized - the first is public employment or
theactual use by the public, and the second is public advantage or benefit. It is also useful
to view the matter as being subject to constant growth, which is to say that as society
advances, its demands upon the individual so increases, and each demand is a new use to
which the resources of the individual may be devoted.
The expropriated property has been shown to be for the continued utilization by the PIA, a
significant portion thereof being ceded for the expansion of the facilities of the Bulacan
State University and for the propagation of the Philippine carabao, themselves in line with
the requirements of public purpose. Respondents question the public nature of the
utilization by petitioner of the condemned property, pointing out that its present use differs
from the purpose originally contemplated in the 1969 expropriation proceedings. The
argument is of no moment. The property has assumed a public character upon its
expropriation. Surely, petitioner, as the condemnor and as the owner of the property, is well
within its rights to alter and decide the use of that property, the only limitation being that it
be for public use, which, decidedly, it is.

2. NO. In insisting on the return of the expropriated property, respondents would exhort on
the pronouncement in Provincial Government of Sorsogon vs. Vda. deVillaroya where the
unpaid landowners were allowed the alternative remedy of recovery of the property there in
question. It might be borne in mind that thecase involved the municipal government of
Sorsogon, to which the power of eminent domain is not inherent, but merely delegated and
of limited application. The grant of the power of eminent domain to local governments
under Republic Act No. 7160 cannot be understood as being the pervasive and all-
encompassing power vested in the legislative branch of government. For local governments
to be able to wield the power, it must, by enabling law, be delegated to it by the national
legislature, but even then, this delegated power of eminent domain is not, strictly speaking,
a power of eminent, but only of inferior, domain or only as broad or confined as the real
authority would want it to be.

Thus, in Valdehueza vs. Republic where the private landowners had remained unpaid ten
years after the termination of the expropriation proceedings, this Court ruled -

The points in dispute are whether such payment can still be made and, if so, in what
amount. Said lots have been the subject of expropriation proceedings. By final and
executory judgment in said proceedings, they were condemned for public use, as part of an
airport, and ordered sold to the government. x x x It follows that both by virtue of the
judgment, long final, in the expropriation suit, as well as the annotations upon their title
certificates, plaintiffs are not entitled to recover possession of their expropriated lots - which
are still devoted to the public use for which they were expropriated - but only to demand
the fair market value of the same.

"Said relief may be granted under plaintiffs' prayer for: `such other remedies, which may be
deemed just and equitable under the premises'."

The Court proceeded to reiterate its pronouncement in Alfonso vs. Pasay City where the
recovery of possession of property taken for public use prayed for by the unpaid landowner
was denied even while no requisite expropriation proceedings were first instituted. The
landowner was merely given the relief of recovering compensation for his property
computed at its market value at the time it was taken and appropriated by the State.

The judgment rendered by the Bulacan RTC in 1979 on the expropriation proceedings
provides not only for the payment of just compensation to herein respondents but likewise
adjudges the property condemned in favor of petitioner over which parties, as well as their
privies, are bound. Petitioner has occupied, utilized and, for all intents and purposes,
exercised dominion over the property pursuant to the judgment. The exercise of such rights
vested to it as the condemnee indeed has amounted to at least a partial compliance or
satisfaction of the 1979 judgment, thereby preempting any claim of bar by prescription on
grounds of non-execution. In arguing for the return of their property on the basis of non-
payment, respondents ignore the fact that the right of the expropriatory authority is far
from that of an unpaid seller in ordinary sales, to which the remedy of rescission might
perhaps apply. An in rem proceeding, condemnation acts upon the property. After
condemnation, the paramount title is in the public under a new and independent title; thus,
by giving notice to all claimants to a disputed title, condemnation proceedings provide a
judicial process for securing better title against all the world than may be obtained by
voluntary conveyance.

Respondents, in arguing laches against petitioner did not take into account that the same
argument could likewise apply against them. Respondents first instituted proceedings for
payment against petitioner on 09 May 1984, or five years after the 1979 judgment had
become final. The unusually long delay in bringing the action to compel payment against
herein petitioner would militate against them. Consistently with the rule that one should
take good care of his own concern, respondents should have commenced the proper action
upon the finality of the judgment which, indeed, resulted in a permanent deprivation of their
ownership and possession of the property.

The constitutional limitation of just compensation is considered to be the sum equivalent


to the market value of the property, broadly described to be the price fixed by the seller in
open market in the usual and ordinary course of legal action and competition or the fair
value of the property as between one who receives, and one who desires to sell, it fixed at
the time of the actual taking by the government. Thus, if property is taken for public use
before compensation is deposited with the court having jurisdiction over the case, the final
compensation must include interests on its just value to be computed from the time the
property is taken to the time when compensation is actually paid or deposited with the
court. In fine, between the taking of the property and the actual payment, legal interests
accrue in order to place the owner in a position as good as (but not better than) the position
he was in before the taking occurred.

The Bulacan trial court, in its 1979 decision, was correct in imposing interests on the zonal
value of the property to be computed from the time petitioner instituted condemnation
proceedings and took the property in September 1969. This allowance of interest on the
amount found to be the value of the property as of the time of the taking computed, being
an effective forbearance, at 12% per annum should help eliminate the issue of the constant
fluctuation and inflation of the value of the currency over time. Article 1250 of the Civil
Code, providing that, in case of extraordinary inflation or deflation, the value of the currency
at the time of the establishment of the obligation shall be the basis for the payment when
no agreement to the contrary is stipulated, has strict application only to contractual
obligations. In other words, a contractual agreement is needed for the effects of
extraordinary inflation to be taken into account to alter the value of the currency.

All given, the trial court of Bulacan in issuing its order, dated 01 March 2000, vacating its
decision of 26 February 1979 has acted beyond its lawful cognizance, the only authority left
to it being to order its execution. Verily, private respondents, although not entitled to the
return of the expropriated property, deserve to be paid promptly on the yet unpaid award of
just compensation already fixed by final judgment of the Bulacan RTC on 26 February 1979
at P6.00 per square meter, with legal interest thereon at 12% per annum computed from
the date of "taking" of the property, i.e., 19 September 1969, until the due amount shall
have been fully paid.

You might also like